CBSE Questions for Class 8 General Knowledge Introduction To Indian Geography Quiz 4 - MCQExams.com

With which country India has the longest international boundary?
  • Nepal
  • Pakistan
  • China
  • Bangladesh
Match List I (tributary) with List II (main rivers) and select the correct answer using the codes given below.
  List I
(Tributary)
List II
(Main Rivers)
 1 Chambal Narmada
 2 Son Yamuna
 3 Manas  Brahmaputra 
Which of the pairs given above is/are correctly matched? 
  • 1, 2 and 3
  • 1 and 2
  • 2 and 3
  • Only 3
Which one of the following is the largest lagoon in India? 
  • Vembanad lagoon
  • Chilika lagoon
  • Pulicat lagoon
  • Kolleru
The river on which the reservoir for Indira Gandhi Canal has been built is __________.
  • Sutlej
  • Jhelum
  • Luni
  • Ravi
If there were no Himalayan ranges, what would have been the most likely geographical impact on India?
Much of the country would experience the cold waves from Siberia.
Indo-Gangetic Plain would be devoid of such extensive alluvial soils.
The pattern of monsoon would be different from what it is at present.
Which of the statements given above is/are correct?
  • Only 1
  • 1 and 3
  • 2 and 3
  • 1,2 and 3
Which one of the following rivers of India does not make a delta?
  • Ganges
  • Godavari
  • Mahanadi
  • Tapti
The monsoon responsible for rains in Jharkhand is ____________.
  • North-East monsoon
  • South-West monsoon
  • Northwester
  • None of these
In India, glacial terraces known as Karewas are found in ___________.
  • Sapt Kosi Valley
  • Jhelum Valley
  • Alaknanda Valley
  • Teesta Valley
Which one among the following rivers does not flow into the Bay of Bengal ?
  • Mahanadi
  • Cauvery
  • Tapti
  • Godavari
River Indus originates from ___________.
  • Hindukush range
  • Himalayan range
  • Karakoram range
  • Kailash range
Which of the following are not fresh water lakes?
Wular Lake                     2. Dal Lake
Sambhar Lake                4. Lonar Lake
Choose the right answer from the options given below.
  • 1 and 3
  • 1 and 2
  • 3 and 4
  • 1, 3 and 4
Consider the following rivers.
Kishenganga
Ganga
Wainganga
Penganga
The correct sequence of these rivers when arranged in the North-South direction is ______________.
  • 1, 2, 3, 4
  • 2, 1, 3, 4
  • 2, 1, 4, 3
  • 1, 2, 4, 3
Chilika Development Authority, the first organisation in the Asian subcontinent to receive Ramsar Wetland Conservation Award for the year 2002, is in ___________.
  • Kerala
  • Odisha
  • Assam
  • Uttarakhand
Consider the following pairs.
Wetlands : Confluence of Rivers
Harike Wetlands : Confluence of Beas and Sutlej
Keoladeo Ghana : Confluence of Banas and Chambal National Park
Kolleru Lake : Confluence of Musi and Krishna
Which of the above pairs is/are correctly matched?
  • Only 1
  • 2 and 3
  • 1 and 3
  • 1, 2 and 3
The river with highest tidal bore in India is ________.
  • Cauvery
  • Mahanadi
  • Hooghly
  • Krishna
The tributary of river Ganga marked X in the given map is which one of the following?

726532_d000b998169945079911a73ab1a4e33c.jpg
  • Gandak
  • Kosi
  • Ghaghra
  • Sone
The correct sequence of the Eastward flowing rivers of the Peninsular India from North to South is ____________.
  • Subarnarekha, Mahanadi, Godavari, Krishna, Pennar, Cauvery and Vaigai
  • Subarnarekha, Mahanadi, Krishna, Godavari, Cauvery and Vaigai
  • Mahanadi, Subarnarekha, Godavari, Krishna, Cauvery, Pennar and Vaigai
  • Mahanadi, Subarnarekha, Krishna, Godavari, Cauvery, Vaigai and Pennar
Which one among the following Indian rivers originates in Maharashtra and flows through Karnataka and Andhra Pradesh to meet the Bay of Bengal in Andhra coast?
  • Mahanadi
  • Cauvery
  • Krishna
  • Godavari
Which one of the following pairs is not correctly matched? 
  • Dam: Govind Sagar ; River: Sutlej
  • Dam: Kolleru Lake ; River: Ganga
  • Dam: Ukai Reservoir ; River: Tapti
  • Dam: Wular Lake ; River: Jhelum
  • Dam: Indira Sagar Dam ; River: Narmada
Consider the following tributaries of river Brahmaputra.
Lohit                     
Tista
Subansiri
Sankosh
Arrange the above rivers from West to East.
  • 2, 4, 3, 1
  • 2, 3, 4, 1
  • 4, 2, 3, 1
  • 3, 1, 2, 4
Which one of the following lakes in India has the highest water salinity?
  • Dal
  • Chilika
  • Wular
  • Sambhar
Consider the following rivers.
Vamsadhara
Indravati
Pranahita
Pennar 
Which of the above are tributaries of Godavari?
  • 1, 2 and 3
  • 2, 3 and 4
  • 1, 2 and 4
  • 2 and 3
Which of the following drainage systems fall into Bay of Bengal?
  • Ganga, Brahmaputra and Godavari
  • Mahanadi, Krishna and Cauvery
  • Luni, Narmada and Tapti
  • Both (a) and (b)
In India, which one of the following states has the largest inland saline wetland?
  • Gujarat
  • Haryana
  • Madhya Pradesh
  • Rajasthan
At which one of the following places do two important rivers of India originate; while one of them flows towards North and merges with another important river flowing towards Bay of Bengal, the other one flows towards Arabian Sea?
  • Amarkantak
  • Badrinath
  • Mahabaleshwar
  • Nasik
Consider the following statements.
Nagarjunasagar project is located on the Krishna River.
The Paithan (Jayakwadi) Hydroelectric project, completed with the help of Japan, is on the river Godavari.
Which of the statements given above is/are true?
  • Only 1
  • Only 2
  • Both 1 and 2
  • Neither 1 nor 2
Krishna river flows through which of the following states? 
  • Maharashtra and Karnataka
  • Maharashtra, Karnataka and Andhra Pradesh
  • Karnataka and Andhra Pradesh
  • Maharashtra, Chhattisgarh and Andhra Pradesh
Consider the following statements.
There are no East flowing rivers in Kerala.
There are no West flowing rivers in Madhya Pradesh.
Which of the statements given above is/are correct?
  • Only 1
  • Only 2
  • Both 1 and 2
  • Neither 1 nor 2
What is the correct sequence of the rivers - Godavari, Mahanadi, Narmada and Tapti in the descending order of their lengths?
  • Godavari - Mahanadi - Narmada - Tapti
  • Godavari - Narmada - Mahanadi - Tapti
  • Narmada - Godavari - Tapti - Mahanadi
  • Narmada - Tapti - Godavari - Mahanadi
Which one of the following rivers does not originate in India? 
  • Beas
  • Chenab
  • Ravi
  • Sutlej
Which one of the following statements is not true?
  • Ghaggar's water is utilised in the Indira Gandhi Canal
  • Narmada rises from Amarkantak region
  • Nizam Sagar is situated on the Manjra river
  • Penganga is a tributary of the Godavari
Which one among the following statements is correct ?
  • In summer season, the duration of day is more in Northern hemisphere.
  • In winter season, the duration of day is more in Northern hemisphere.
  • In summer season, the duration of day is less in Northern hemisphere.
  • In winter season, the duration of day is more in Southern  hemisphere.
Which one of the following pairs is not correctly matched? 
River : City
  • Gomti : Lucknow
  • Saryu : Ayodhya
  • Alaknanda : Badrinath
  • Narmada : Satna
Which of the following is the longest river in India?
  • Ganga
  • Brahmaputra
  • Yamuna
  • Godavari
In India, 'Yarlung Zangbo' river is known as ____________.
  • Ganga
  • Indus
  • Brahmaputra
  • Mahanadi
A devastating Cloud Burst swept over Leh in AugustWhich one of the following statements with regard to Cloud Burst is not correct?
  • Cloud Burst is a localised weather phenomenon representing highly concentrated rainfall over a small area in a short duration of time.
  • Cloud Burst occurs due to upward movement of moisture-laden air with sufficient rapidity to form cumulonimbus clouds.
  • Cloud Burst occurs only in hilly areas.
  • There is no satisfactory technique till now for predicting Cloud Burst.
An important river of the Indian desert is ___________.
  • Luni
  • Narmada
  • Krishna
  • Beas
Which one of the following rivers forms an estuary?
  • Narmada
  • Cauvery
  • Krishna
  • Mahanadi
The South-West monsoons end in India by ______________.
  • December
  • October
  • November
  • July
The January isotherm taken as a basis for dividing India into tropical and subtropical zones is ______________.
  • $$21^0 C$$
  • $$180^0 C$$
  • $$12^0 C$$
  • $$15^0 C$$
The alpine vegetation in the Western Himalayas is found only upto a height of 3000 metres while in the Eastren Himalayas it is found upto a height of 4000 metres. The reason for this variation in the same mountain range is that ____________.
  • Eastern Himalayas are higher than Western Himalayas.
  • Eastern Himalayas are nearer to the equator and sea coast than Western Himalayas.
  • Eastern Himalayas get more monsoon rainfall than the Western Himalayas.
  • Eastern Himalayan rocks are more fertile than the Western Himalayan rocks.
Which one of the following mountain ranges is spread over only one state in India?
  • Aravalli
  • Satpura
  • Ajanta
  • Sahyadri
In which one of the following islands of India an active volcano is found?
  • Car Nicobar Island
  • Nancowry island
  • Barren Island
  • Maya Bunder Island
The type of natural vegetation in India varies due to variation in ________.
1) the amount of rainfall
2) soil types
3) mean annual temperature
4) altitude 
Select the correct answer using the codes given below.
  • 1 and 2
  • 1, 2 and 3
  • 2, 3 and 4
  • 1, 3 and 4
What is the name of the strait where 'Adam's bridge' is situated?
  • Bering Strait
  • Cook Strait
  • Palk Strait
  • Taiwan Strait
Which one of the following is the oldest mountain range in India?
  • Himalayas
  • Aravalli
  • Satpura
  • Nilgiri
Tree cover is maximum in _________.
  • East Deccan
  • Northern Plains
  • West Coast
  • East Coast
Which of the following statements with regard the expansion of the desert in Rajasthan is / are correct ?
1) The predominant wind direction in Rajasthan is North-East to South -West , so the tendency for desertification has been more in that direction.
2) The Thar desert is dominantly a monsoon driven where wind erosion is a major problem during the summer months while the Aravalli range is a major barrier in the spread of the desert , rampant mining creating cuts in the range is leading to the spread of the desert.
Select the correct answer using the codes given below :-
  • Only 1
  • Only 2
  • Both 1 and 2
  • Neither 1 nor 2
Consider the following statements regarding the Western coastal plains of India.
The Western coastal plains are known as Konkan from Daman to Goa.
The plains are narrower in the North and wider in the South.
Which of the above statement(s) is/are correct?
  • Only 1
  • Only 2
  • Both 1 and 2
  • Neither 1 nor 2
The direction of monsoons of the Arabian Sea branch is from __________.
  • North-West
  • Southern tip to West
  • South-East
  • None of these
0:0:1


Answered Not Answered Not Visited Correct : 0 Incorrect : 0

Practice Class 8 General Knowledge Quiz Questions and Answers